Why not B
Why couldn't the answer be B. Maybe the host, died then the next day the outbreak occurs. Or ...
Nativeguy on August 1 at 07:10PM
  • June 2011 LSAT
  • SEC1
  • Q7
1
Reply
Why not B
Why couldn't the answer be B. Maybe the host, died then the next day the outbreak occurs. Or ...
Nativeguy on August 1 at 07:10PM
  • June 2011 LSAT
  • SEC1
  • Q7
1
Reply
I don't see the flaw
I am mapping this stimulus out as: P1. Change --> Motive P2. Unpleasant criticism --> Motive ...
AndrewArabie on November 6, 2022
  • June 2011 LSAT
  • SEC1
  • Q25
2
Replies
Most strongly supported?
I thought "most strongly supported" is a strengthen question? How often does the LSAT use "Most s...
Tyler808 on June 1, 2022
  • June 2011 LSAT
  • SEC1
  • Q12
1
Reply
Confused on why the correct answer is D.
I wanted to pick D but the stimulus said that the University "found that he had not" falsified th...
kaylarussell on October 15, 2020
  • June 2011 LSAT
  • SEC1
  • Q11
1
Reply
Confused on why the correct answer is D.
I wanted to pick D but the stimulus said that the University "found that he had not" falsified th...
Jason on October 15, 2020
  • June 2011 LSAT
  • SEC1
  • Q11
1
Reply
Choice C
Can someone explain why answer choice C is wrong?
shafieiava on October 7, 2020
  • June 2011 LSAT
  • SEC1
  • Q23
2
Replies
Why is B wrong?
I understand why A is correct, can someone please explain why B is wrong?
djayasinghe on October 1, 2020
  • June 2011 LSAT
  • SEC1
  • Q25
1
Reply
Clarification
I don't understand why D is correct over B
tselimovic on October 1, 2020
  • June 2011 LSAT
  • SEC1
  • Q14
2
Replies
Could someone please help explain this question
I don't understand what this question is asking/which method I should use.
liwenong28 on September 18, 2020
  • June 2011 LSAT
  • SEC1
  • Q6
1
Reply
Please Help!
@lsatmax I'm sort of confused between why answer choice (A) is correct and answer choice (C) is i...
Lauren-Au on September 2, 2020
  • June 2011 LSAT
  • SEC1
  • Q25
2
Replies
D vs. E
Can you explain why D is wrong in comparison to E?
mosie on August 25, 2020
  • June 2011 LSAT
  • SEC1
  • Q8
1
Reply
Help diagramming
I got the initial premise backwards when I diagrammed and it led to me selecting an incorrect ans...
Tegan on August 19, 2020
  • June 2011 LSAT
  • SEC1
  • Q20
1
Reply
D Versus E?
I narrowed it down to these two alternatives and ultimately chose the correct answer, but I'm sti...
JoshG on July 7, 2020
  • June 2011 LSAT
  • SEC1
  • Q14
3
Replies
Why is E wrong?
I am not seeing how the first part of this argument cannot be considered a generalization. Thanks!
mahosmar on June 6, 2020
  • June 2011 LSAT
  • SEC1
  • Q17
1
Reply
Could someone please explain this question?
Could someone please explain this question? Thanks
jingjingxiao11111@gmail.com on May 31, 2020
  • June 2011 LSAT
  • SEC1
  • Q9
1
Reply
WHY NOT A
Hi LSAT MAX, can you please explain why A is incorrect? Thanks!
Yuer-Wang on March 25, 2020
  • June 2011 LSAT
  • SEC1
  • Q8
1
Reply
Answer Explanation
Hi there, can someone explain why D is the right answer? Thanks!
M-Howard on January 16, 2020
  • June 2011 LSAT
  • SEC1
  • Q4
3
Replies
Mapping
Is it possible for an instructor to show how you might map/diagram this question to keep the prin...
shafieiava on December 14, 2019
  • June 2011 LSAT
  • SEC1
  • Q22
1
Reply
Question type? Answer choice E?
Is this a strengthen with sufficient premise question? Is the correct answer identifying the jump...
shafieiava on December 13, 2019
  • June 2011 LSAT
  • SEC1
  • Q19
1
Reply